User Avatar
crivera772
Joined
Apr 2025
Subscription
Free
PrepTests ·
PT134.S2.Q16
User Avatar
crivera772
Tuesday, Jan 30 2018

Beautiful

PrepTests ·
PT117.S2.Q14
User Avatar
crivera772
Friday, Dec 22 2017

Shouldn't the left part of the likely arrow be a disjunction of negated literals rather than a conjunction of negated literals?

PrepTests ·
PT117.S3.Q9
User Avatar
crivera772
Sunday, Oct 22 2017

Most strongly supported questions ask for an inference, for which the premises act as support. The strength of support tends towards validity, without actually being valid.

Answer choice D is strongly supported, however, the support, along with answer choice D, does not make the argument valid. Answer choice D relies on the assumption that denying children the responsibility to vote amounts to denying children the right to vote.

PrepTests ·
PT119.S1.P4.Q22
User Avatar
crivera772
Saturday, Oct 21 2017

This passage was difficult to understand, even on the second reading for BR; does one just get better at understanding hard passages with more practice?

PrepTests ·
PT118.S1.Q15
User Avatar
crivera772
Tuesday, Oct 17 2017

D is wrong because its pith regards a tacit assumption about private neighborhoods, which presumably are built with private funds. However, the pith of the stimulus regards public neighborhoods, and public funds.

D is subtly out of scope, and thus does not describe why the premises fail to support the conclusion.

PrepTests ·
PT118.S1.Q8
User Avatar
crivera772
Tuesday, Oct 17 2017

How do we know that personal relationships additionally require trust?

PrepTests ·
PT118.S1.Q4
User Avatar
crivera772
Tuesday, Oct 17 2017

I got trapped. Is answer choice (B) wrong because the statement: "Analysts would stand to gain if this was true" is not the pith of the columnist's argument?

PrepTests ·
PT118.S4.Q23
User Avatar
crivera772
Tuesday, Oct 17 2017

I chose (E); but considering (A), it is not factually incorrect. Is it wrong because (A) implies the absence of the main conclusion?

PrepTests ·
PT126.S4.Q17
User Avatar
crivera772
Saturday, Dec 16 2017

Refutation by Parallel Reasoning:

Couch Potato: Some of my opponents have argued that it is necessary for me to have a low body-fat ratio in order to live a healthier and more fulfilling life. My opponents should instead argue on the main cause of my not wanting to go to the gym: I'm lazy and unmotivated. Thus, it is unwarranted for me to have a low body-fat ratio in order to live a healthier and more fulfilling life.

In both arguments, a certain ends is proposed by some opponents; the means to obtain those ends is presumed by the author to be only such and such, and the author further assumes that the opponents in question are also holding the same presumption that the means to achieve said ends is only such and such. There could be other means to obtain those ends, for instance, low body fat can be achieved without having to go to the gym at all, one can restrict their calorie intake or do intermittent fasting (abs are made in the kitchen), and still successfully achieve a low body-fat percentage.

User Avatar
crivera772
Monday, Oct 16 2017

You better be watching Super J.Y. Jiren is a badass. He's so zen he'd get a 180 on the LSAT; and he'd be writing it while he was fighting Goku.

User Avatar
crivera772
Saturday, Oct 14 2017

My > @ said:

Wait, what's this post trying to get at? It sounds like you're trying to convince us that you're not alt-right or liberal (although those are by no means equatable). Unless you wrote this as your PS, why would the admissions committee know this about you or care about this thought process? If you're concerned about the effect of your politics, just don't write about them.

Good point.

User Avatar

Wednesday, Oct 11 2017

crivera772

Political views

Hi,

How does one's political views affect one's chances at admission, especially to elite schools which tend to lean left? I wouldn't say I'm a liberal, by today's standards of what constitutes left at least; if one defines "liberal" as "one who values liberty", then yes I am a liberal. However, I try to understand both sides of the political spectrum without being wedded to any particular ideology. By no means am I what many have called the "alt-right", however, there are some views that they hold that I am sympathetic towards; in particular, I believe that it is necessity to guard the freedom of speech. There are, of course, reasons for me to be sympathetic towards this idea: language evolves. To legislate on what constitutes "appropriate" speech is to attempt to directly and consciously control the evolution of language. If I were to reject this notion, I would have to reject the liberal value of liberty, which is distinct from license. Thus, to remain a liberal, it seems that I must defend, what is now considered to be, a conservative ideal.

In short, I am a centrist. But it seems that being a centrist, especially in this politically charged climate, would hurt my chances. Am I correct in being concerned?

PrepTests ·
PT112.S3.Q11
User Avatar
crivera772
Wednesday, Oct 11 2017

Why is E not a slight strengthener? It seems relevant. One could argue that the situation presented in choice E is not necessarily a representative sample of the leaf preferences of all other insects that eat leaves, but on what basis would one have to make this assumption? One would have to assume more than is warranted in the argument to make say that the situation is unrepresentative in order to eliminate answer choice E.

If I were taking this test, under timed conditions, I wouldn't be able to eliminate for this reason. Am I explaining it to myself correctly? Or is there another, perhaps stronger, reason to eliminate E?

PrepTests ·
PT111.S4.Q21
User Avatar
crivera772
Sunday, Apr 08 2018

For answer choice E, one could also say that it is incorrect for saying that the social system would be undermined rather than the values of that social system.

Confirm action

Are you sure?